pls help:( algebra 1 btw


17(2 + 12g) + 14

Answers

Answer 1

Answer:

204g + 48

Step-by-step explanation:

simplifying the expression

Answer 2
Answer: 204g+48


Step 1: Rewrite equation

To start off, let’s rewrite the equation to better understand the problem we are working with.

17(2+12g)+14

Step 2: Distribute

In this type of equation, we will not find an exact answer. Rather, we will simplify the equation by distribution and combining like terms. Let’s start by distributing. To do so, multiply the numbers outside of the parentheses with the numbers on the inside of the parentheses. This will be 17•2 and 17•12g. Let’s do so now.

17(2+12g)+14
34+204g+14

Step 3: Combine like terms

As the final step, we need to combine like terms. This means we add/subtract terms that share the same variable, or lack of. In this case, we need to add 34 and 14 together. Let’s do this now.

34+204g+14
204g+48

This is your final answer. 204g+48 is the new equation. Hope this helps! Comment below for more questions.

Related Questions

Can someone help me with this question

Answers

Vas happening?
You break it down
3 ^5 is 3 x 3 x 3 x 3 x 3 = 729
4 ^ 5 is 4 x 4 x 4 x 4 x 4 = 1024
729 times 1024 = 746,496
12 ^ 10 is 12 x 12 x 12 x 12 x 12 x 12 x 12 x 12 x 12 x 12 = 6.19174e10
So the answer is incorrect

Answer:

Kenji is wrong

Step-by-step explanation:

3⁵ · 4⁵ = (3 · 4)⁵ = 12⁵

but 12¹⁰ ≠ 12⁵

so , kenji is wrong

find perimeter of the semicircular region​

Answers

Answer:

4. 87.96 yd

5. 47.12 ft

6. 34.56 cm

Step-by-step explanation:

4. diameter(d) = 28 yd,

perimeter = πd = 87.96 yd (rounded to 2 dp)

5. radius(r) = 7.5 ft

perimeter = 2πr = 47.12 ft (rounded to 2 dp)

6. diameter (d) = 11 cm

perimeter = πd = 34.56 cm

Answered by GAUTHMATH

Formula to find the perimeter of a semicircular region:

if radius given: [tex]P=2r[/tex] +  [tex]\pi r[/tex]  {r = radius}

if diameter given: [tex]P=d+\pi r[/tex] {d = diameter}

Take [tex]\pi[/tex] as 3.14

Q4.

Diameter = 28 yd

Radius = 28/2 = 14 yd

[tex]P=28+\pi (14)[/tex]

[tex]P=28+(3.14)(14)[/tex]

[tex]P=28+14.96[/tex]

[tex]P=42.96[/tex] yd

Q5.

Radius = 7.5 ft

Diameter = 7.5 × 2 = 15 ft

[tex]P=2(7.5)+\pi (7.5)[/tex]

[tex]P=2(7.5)+(3.14)(7.5)[/tex]

[tex]P=15+23.55[/tex]

[tex]P=38.55[/tex] ft

Q6.

Diameter = 11 cm

Radius = 11/2 = 5.5 cm

[tex]P=11+\pi (5.5)[/tex]

[tex]P=11+(3.14)(5.5)[/tex]

[tex]P=11+17.27[/tex]

[tex]P=17.27[/tex] cm

I hope this helps...

Have a great day ahead :)

Find the size of each of the unknown angles. Help me plz​

Answers

Answer:

2a+15 = 125(being alternate angle)

or,2a = 125-15

or,2a = 110

or,a = 110÷2

,a = 55

again,2a+15+b+30=180(Being co-interior angle)

or,2×55+15+b+30=180

or,110+15+30+b=180

or,155+b=180

or,b=180-155

Therfore,b=25

simplyfy using appropiate properties 1/4×2/5+-1/6×3/2+3/7×2/5 ​

Answers

Answer:

-1/35

Step-by-step explanation:

* means multiply

because of pemdas you should do three multiplication parts first

(1/4×2/5)+(-1/6×3/2)+(3/7×2/5)

1/20 - 3/12 + 6/35

3/60 - 15/60 + 6/35

-12/60 + 6/35

-1/5 + 6/35

-7/35 + 6/35

-1/35

Answer:

3/140

Step-by-step explanation:

1/4 × 2/5 + (-1/6) × 3/2 + 3/7 × 2/5 ​=

= 2/20 - 3/12 + 6/35

**********************************************

We need the LCD of 20, 12, 35.

20 = 2^2 * 5

12 = 2^2 * 3

35 = 5 * 7

LCD = 2^2 * 3 * 5 * 7 = 420

**********************************************

= 42/420 - 105/420 + 72/420

= 9/420

= 3/140

3
49:51
What is the solution to the system that is created by the equation Y =-X+6 and the graph shown below?
10
8
6
4
-10-8-6-44
2
4 6
8 10 x
4
46
-8
10
O (-8,-4)
O (-4,-2)
(4.2)
(6.3)
Mark this and retum
Save and Exit
Next
Submit

Answers

If you draw the graph of y = -x + 6 on the above graph the solution to system will be the coordinates of the point of intersection.
the equation of the line drawn above is y = 0.5x

so you can also find the solution by solving the system
y = 0.5x
y = -x + 6

0.5x = -x + 6
1.5x = 6
x = 6/1.5 = 4

and y = 0.5*4 = 2

The answer is (4, 2) - the third choice.

Select the equivalent expression.
(6-4.8-7)+"=?

Answers

6-4 is 2,8-7 is 1 2x1 is 2

:. In the diagram below, AC is congruent to CE and D is the midpoint of CE. If CE = 10x + 18, DE = 7x - 1, and BC = 9x - 2, find AB.​

Answers

Answer:

25

Step-by-step explanation:

Since AC is similar to CE, we can say that they have equal lengths. We are given CE, DE, and BC. To solve this, we can solve for the length of CE (equal to AC) and then subtract that from BC.

To solve for CE, we are given DE and CE. We know that DE is 1/2 of CE because D is the midpoint of CE. Therefore,

7x-1 = 1/2(10x+18)

expand

7x-1 = 5x  + 9

add 1 to both sides to separate the 7x

7x = 5x + 10

subtract 5x from both sides to separate the x

2x = 10

divide both sides by 2 to separate the x

x=5

Therefore, DE = 7(5)-1 = 34 and CE = 10(5) + 18 = 68 = AC.

Using x=5, we know that BC  = 9(5) -2 = 43

Therefore, AB = AC-BC = 68-43 = 25

D is the midpoint of CE, so if you draw a line with those three points, it'll look like C-D-E.

Since DE = 7x-1, which also means CD = 7x-1.

CD + DE = CE, so (7x-1)+(7x-1) = 10x+18.

Therefore, x = 5 and CE = 68.

Since AC is congruent to CE, AC = 68.

Assuming the point B is somewhere between AC.

Since BC = 9x-2 and x = 5, which means BC = 43.

AC - BC = AB, so 68 - 43 = 25.

Therefore, AB = 25

A class conduct an experiment in biology. They place a number of 1 m by 1 m square grids on the playing field and count the number of plants in each grid. The results obtained are given below.

6 3 2 1 3 2 1 3 0 1
0 3 2 1 1 4 0 1 2 0
1 1 2 2 2 4 3 1 1 1
2 3 3 1 2 2 2 1 7 1
(a)
Calculate the mean number of plants.

Answers

Step-by-step explanation:

their mean is....

6+3+2+1+3+2+1+3+0+1/10=2.1

3+2+1+1+4+0+1+2+0/10=1.4

1+1+2+2+2+4+3+1+1+1/10=1.8

2+3+3+1+2+2+2+1+7+1/10=2.4

the means are 2.1,1.4,1.8,2.4

the average mean is

21+14+18+24/40

32+45/40

77/40=1.925

the average mean is 1.925

f(b) = 7b^3 +8b and g(b) = b ^2+ b - 10. What is f(b)-g(b)?

Answers

Answer:

[tex]{ \bf{f(b) - g(b) : }} \\ { \tt{ = ( {7b}^{3} + 8b) - ( {b}^{2} + b - 10)}} \\ = ( {7b}^{3} - {b}^{2} + 7b + 10)[/tex]

Answer:

[tex]f(b) - g(b) = 7b^3 -b^2 + 7b +10[/tex]

Step-by-step explanation:

[tex]f(b) = 7b^3 + 8b \\\\g(b) = b^2 + b - 10\\\\f(b) - g(b) = (7b^3 + 8b ) - ( b^2 + b -10)[/tex]

               [tex]= 7b^3 + 8b - b^2 - b + 10\\\\=7b^3 - b^2 +7b + 10[/tex]

Find the total number of outcomes in each experiment. Write your answers on a sheet of paper.

1. tossing a coin
2. tossing 3 coins
3. rolling a die 10 times
4. rolling two dices
5. pressing a number key on a calculator
6. picking a card from a regular deck of cards
7. choosing a letter from the English alphabet
8. choosing a letter from the word OUTCOME
9. choosing a letter from the word PREDICTION
10. picking a crayon from a box with 36 crayons of different colors

Answers

Answer:

Step-by-step explanation:

In each option you need to find the number of outcomes of a single event and then multiply that by the number of times that event takes place.

1. 2 outcomes (heads and tails)

2. 6 outcomes (2 outcomes * 3 tosses)

3. 60 outcomes (6 outcomes per die * 10 rolls)

4. 12 outcomes (6 outcomes per die * 2 rolls)

5. 10 outcomes (10 numbers on the pad)

6. 52 outcomes (52 cards in a regular deck)

7. 32 outcomes (32 letters in the alphabet)

8. 7 outcomes (7 letters to choose from)

9. 10 outcomes (10 letters to choose from)

10. 36 outcomes (36 crayons to choose from)

HELP ASAP 10 POINTS AND BRAINLIST HURRY AND WILL GIVE 5 STAR AND A THANKS

Answers

A= 18 units squared
A of square = 9 units squared
A of triangles = 4.5 each unites squared

Help and explain please and thank you !!!!!!!

Answers

Answer:

[tex]q = 3[/tex]

Step-by-step explanation:

[tex]3q−2=7[/tex]

[tex]3q=7+2[/tex]

[tex]3q=9[/tex]

[tex]q = \frac{9}{3} [/tex]

[tex]q = 3[/tex]

Hope it is helpful....
Option C is correct. The other person is wrong. They forgot the two lines flip the equation. This is an equation that can be solved. Look at the picture below to understand the simplified steps. Give brainiest plz. I have never had it before. Thx.

rization
Vhat is the prime factorization of 180?
Select a composite number to break into factors. Continue
factoring until all factors are prime.
22•2•3•3.5
2.2.45
2.3.3.5
480
2.9.5

Answers

Answer:

5 3 3 2 2

Step-by-step explanation:

180

90 2

45 2 2

15 3 2 2

5 3 3 2 2

Answer:

180 = 5 × 3² × 2²

Step-by-step explanation:

Starting with 180, the prime factors are in bold

180 = 90 × 2

      = 45 × 2 × 2

      = 15 × 3 × 2 × 2

      = 5 × 3 × 3 × 2 × 2

Then

180 = 5 × 3 × 3 × 2 × 2 = 5 × 3² × 2²

Can someone help me explain this question? :( I have to record myself answering it and send it to my teacher

What is the effect on the graph of f(x) if it is changed to f(x) + 7, f(x + 7) or 7f(x)?

Answers

Step-by-step explanation:

The function f(x)+7 will shift the parent function up 7 units.

The function f(x+7) will shift the parent function to the left 7 units.

The function 7f(x) will stretch vertically by a factor of 7.

If
5
3 cosα = in the first quadrant, what does cot α
equal?

Answers

Answer:

5/4

Step-by-step explanation:

To Find :-

cot a .

Solution :-

By question ,

=> cos a = 5/3 = b/h

=> p = √ 5² - 3² = √ 25 -9 = 4

Therefore ,

=> cot a = b/p = 5/4

On a coordinate plane, line D F goes through points (negative 1, negative 3) and (2, 3). Point G is at (negative 4, negative 4).

Answers

The line passes through the y-axis at point (0,4).

Step-by-step explanation:

On a coordinate plane, line DF goes through points (-1,-3) and (2,3).

So, the slope of the line DF is \frac{- 3 - 3}{- 1 - 2} = 2

−1−2

−3−3

=2

Then the straight line which is parallel to DF will be given by

y = 2x + c.

Where, c is any constant and we have to evaluate it if this line passes through the point (-4,-4).

So, - 4 = 2(- 4) + c

⇒ c = 4

So, the straight line which is parallel to DF and passes through the point (-4,-4) is y = 2x + 4.

Now, putting x = 0, we get, y = 4.

Therefore, the line passes through the y-axis at point (0,4). (Answer)

Answer:

The answer would be D

Step-by-step explanation:

There are 103 pounds of wood pieces in a bag. Each wood piece weighs 53 pounds. How many wood pieces are there in the bag?

Answers

Answer:

Approximately 2

Step-by-step explanation:

103 divided by 53 is 1.94

Round 1. 94 up to a whole number is approximately 2 wood pieces.

Express each of the following quadratic functions in the form of f(x) = a (x - h)²+ k.Then,state the minimum or maximum value,axis of symmetry and minimum or maximum point. (a) f(x) = -2x² + 7x + 4.
Pls help me ans this question,its urgent!!
I'll mark u as the brainliest!​

Answers

ok this is answer any questions

If a^2 -b^2 =12 and a-b=4 what is the bay of a +b

Answers

Answer:

a+b = 3

Step-by-step explanation:

a = b+4

a^2 = (b+4)(b+4) = b^2 + 8b +16=-b^2 = 12

8b=-4

b = - 1/2

a = 3.5

What is the measure of arc BC?
А
o
50°
25
E
C
D
pls help if u can:)

Answers

Answer:

arc BC = 100°

Step-by-step explanation:

The inscribed angle BEC is half the measure of its intercepted arc BC , then

arc BC = 2 × 50° = 100°

Which descriptions from the list below accurately describe the relationship between QRS and TUV? check all the apply.

Answers

Answer:

The triangles are similar.

Step-by-step explanation:

They are similar because they have the same angles. They may be the same size, but it would be best to refer to whatever definition you were given.

The correct descriptions from the list accurately describe the relationship between QRS and TUV is,

⇒ Similar

What is mean by Triangle?

A triangle is a three sided polygon, which has three vertices and three angles which has the sum 180 degrees.

We have to given that;

Two triangles are shown in figure.

Here, All the corresponding angles are same in both triangles.

And, The ratio of their corresponding sides are,

In triangle TUV;

⇒ 90 / 108 = 30 / 36

In triangle QRS;

⇒ 30 / 36

Hence, The correct descriptions from the list accurately describe the relationship between QRS and TUV is,

⇒ Similar

Learn more about the triangle visit;

brainly.com/question/1058720

#SPJ7

Which equation in slope-intercept form represents a line that is parallel to y=-4x-5 and passes through the points (0,0)?

Answers

Answer:

y = 0x

Step-by-step explanation:

y=-4x-b

0 = -4(0) + b

0 = 0 + b

0

In the equation 17x2 = 12x, the value of c is:
O 0
O 12
O 17

Answers

The value of c is 0.

By question,

17x² = 12x

or,(17x-12):x=0

or,17x - 12 = 0#

For some tips,

In the equation 17x2 = 12x,Rewrite in factored formMove terms to the left sideCreate separate equations

HENCE PROVED ##

Kala’s family is shopping for a large screen TV. The TV they decided on has a retail price of $1,000 at four stores. The state sales tax is 5%. Each store is offering a different sales promotion, as listed in the table.

Store
Promotional Offer
A
$80 rebate
B
10% discount on price
C
Store pays all sales tax
D
$40 rebate with store paying half of sales tax


Which store has the best deal?
store A
store B
store C
store D

Answers

Answer:

B-10% discount on price

Step-by-step explanation:

A - $1050 - 80 = $970

B - $900 + $45 = $945

C - $1000

D - $1025 - $985

Answer:

store b

Step-by-step explanation:

what is the volume of the figure. answer plz.

Answers

Answer:

Volume: 37.692 cm3

Step-by-step explanation:

1) 1/3 x 3.14 x 3 power of 2 x 4

2) 1.047 x 9 x 4

3) 37.692 cm3

Answer:

[tex]Volume ~of~cone=\pi /3r^{2} h[/tex]

[tex]r=3cm[/tex]

[tex]h=4 cm[/tex]

[tex]volume=\pi /3\times3^{2} \times4[/tex]

[tex]=12\pi =12\times3.14[/tex]

[tex]=37.68~ cm^{2}[/tex]

-----------------------

hope it helps...

have a great day!!

please help i have to resit math final so bare with me


help me with this equation : x^2 - 7 = 0 IN QUADRATIC EQUATION

PS. 1st one to answer gets a brainly crown :)

Answers

I hope this is the correct answer

Choices:

One-sixth cups
Five-sixths cups
1 and two-thirds cups
1 and 5 / 6 cups

Answers

Answer:

Patel will require more orange juice = [tex]\frac{371}{660}[/tex] cups

Step-by-step explanation:

Patel needs the orange juice for his family = 3 cups

He needs more orange juice = 3 cups - Sum of juice squeezed from different oranges

[tex]=3-(\frac{13}{15}+\frac{1}{5}+\frac{9}{20}+\frac{5}{11}+\frac{7}{15})[/tex]

L.C.M. of the denominators = 660

[tex]=3-\frac{572+132+297+300+308}{660}[/tex]

[tex]=3-\frac{1609}{660}[/tex]

= [tex]\frac{1980-1609}{660}[/tex]

= [tex]\frac{371}{660}[/tex]

Therefore, amount of orange juice required more = [tex]\frac{371}{660}[/tex] cups

What are the factors of 2x2 + 3x - 54? Select two options.
2x - 9
2x - 6
O 2x + 6
OX-6
O x + 6

Answers

Step-by-step explanation:

Therefore, the factors are x + 6 and 2x − 9.

The factors of the polynomial are (2x-9)(x+6)

What is factor?

Factors are the numbers that can divide a number exactly. Hence, after division, there is no remainder left. Factors are the numbers you multiply together to get another number. Thus, a factor is the divisor of another number.

Given is a polynomial 2x²+3x-54, we need to find its factors,

So,

Factors :-

= 2x²+3x-54

= 2x²+12x-9x-54

= 2x(x+6)-9(x+6)

= (2x-9)(x+6)

Hence, the factors of the polynomial are (2x-9)(x+6)

Learn more about factors, click;

https://brainly.com/question/14209188

#SPJ5

In a triangle ABC, a=4 cm, b=3 cm and angleC=30°, find the area of triangle ABC.
a. 6
b. 1.5
c. 3
d. 3 root 3 ​

Answers

the correct answer is c

What is the answer to this question

Answers

Answer:

B - Congruent : SAS

Step-by-step explanation:

We are given that there are two sides that are congruent and one angle is congruent and it is in the order Side Angle Side.

Other Questions
Please help ASAP !!! Thank you ! Skyline Florists uses an activity-based costing system to compute the cost of making floral bouquets and delivering the bouquets to its commercial customers. Company personnel who earn $180,000 typically perform both tasks; other firm-wide overhead is expected to total $70,000. These costs are allocated as follows: Bouquet Production Delivery Other Wages and salaries 60% 30% 10%Other overhead 50% 35% 15%Riverside anticipates making 20,000 bouquets and 4,000 deliveries in the upcoming year. The cost of wages and salaries and other overhead that would be charged to each bouquet made is closest to:a. $12.50.b. $7.15.c. some other amount.d. $8.75.e. $13.75. Select the correct answer. The product of two numbers is 21. If the first number is -3, which equation represents this situation and what is the second number? A. The equation that represents this situation is x 3 = 21. The second number is 24. B. The equation that represents this situation is 3x = 21. The second number is 7. C. The equation that represents this situation is -3x = 21. The second number is -7. D. The equation that represents this situation is -3 + x = 21. The second number is 18. A and \angle BB are vertical angles. If m\angle A=(5x-9)^{\circ}A=(5x9) and m\angle B=(8x-30)^{\circ}B=(8x30) , then find the value of x Which property of addition is shown in the equation below?a+bi+0+0i = a + bicommutative propertyinverse propertyidentity propertyassociative property please help, 3 spanish words for each question plz help ASAP with explanation Could you pleaseeee help me i need some help i need it before tomorrow at 4:00pm How did Watergate affect American public opinion?Help please!! How can I pass the variable argument list passed to one function to another function. Find an equivalent ratio for the proportional relationship. PQ restaurant offers 5 chicken rolls for $6. Which point represents the unit rate? Which sentence best avoids bias?A. If there is an engineer on staff, he likely knows me.B. The girls from accounting know I'm a good boss.C. I have extensive experience in women's health.D. Every Veterans Day, I support the men overseas. A buyer has decided to offer $335,000 for a home that she really likes. The bank will loan her 80% of the purchase price for 30 years at 5% interest. What will be the amount of her principal and interest payment if the requirement is $5.68 per thousand of the loan amount? If copies of all your computer data are stored on four independent hard disk drives, what is the probability that during a year, you can avoid catastrophe with at least one working drive? With four hard disk drives, the probability that catastrophe can be avoided is Based on this model, households earn income when firms __________ purchase goods and services ___________in markets for factors of production. Suppose Caroline earns $625 per week working as jewelry appraiser for Classy's Jewelry Store. She uses $10 to get her car washed at Spotless Car Wash. Spotless Car Wash pays Antonio $275 per week to wash cars. Antonio uses $150 to purchase a necklace from Classy's Jewelry Store. Identify whether each of the following events in this scenario occurs in the market for factors of production or the market for goods and services. Event Market for Factors of Production Market for Goods and Services Caroline spends $10 to get her car washed. Antonio spends $150 to purchase a necklace from Classy's Jewelry Store. Antonio earns $275 per week working for Spotless Car Wash. Which of the elements of this scenario represent a flow from a household to a firm? This could be a flow of dollars, inputs, or outputs. i. The car wash Caroline receives ii. The $275 per week iii. Antonio earns working for Spotless Car wash The $150 Antonio spends to purchase a necklace from Classy's Jewelry Store The actual economy is more complicated than the one illustrated in the previous circular-flow diagram of a simple economy True / False An automatic stabilizer:________ a) increases inflationary pressure during expansions. b) increases the drop in disposable income during recessions and increases the jump in disposable income during expansions. c) increases tax revenue relative to government spending throughout the business cycle. d) decreases tax revenue relative to government spending throughout the business cycle. e) reduces the drop in disposable income during recessions and reduces the jump in disposable income during expansions. Joseline waited until December 12, 2019, to file her 2018 Form 1040 return. She did not request an extension. Her balance due for 2018 is $461. What is her failure to file penalty Required: Monson sells 15 units for $20 each on December 15. Assume the periodic inventory system is used. Determine the costs assigned to ending inventory when costs are assigned based on the weighted average method. (Amounts to be deducted should be indicated with a minus sign. Round cost per units to 2 decimals.) Twice a number minus 25 is less than 89. Translate it into an inequality and find the solution